5/21/2011

[繪圖] 塗鴉

Jesus love you

by Chung-Han Hsieh
工具:蠟筆

5/12/2011

[數學分析] 逐點收斂與均勻收斂(3) - Differentiation property

如果我們手邊有一個 均勻收斂的 函數 sequence $\{f_n \}$ 且假設此數列可微,我們想知道均勻收斂是否能給我們一些關於此函數sequence 微分 $\{ f_n' \}$ 的一些關聯?

首先看個例子:

Example:

\[
f_n(x) := \frac{\sin nx}{\sqrt{n}}, \;\; (x \in \mathbb{R}, \;n=1,2,3,...)
\]試回答下列問題:
1. 函數是否逐點收斂(converges pointwise)?
2. 是否均勻收斂(converges uniformly)?
3. 此函數sequence 導數 $f_n'(x)$ 為何?
4. 此函數sequence 的導數  $\{ f_n'\}$是否逐點收斂?
5. 此函數sequence 的導數  $\{ f_n'\}$是否均勻收斂?
Solution
1. 首先檢驗是否逐點收斂
給定 $x \in \mathbb{R}$,我們可知道當 $n \rightarrow \infty$ 函數 sequence $\{f_n \}$為
\[\mathop {\lim }\limits_{n \to \infty } {f_n}\left( x \right) = \mathop {\lim }\limits_{n \to \infty } \frac{{\sin nx}}{{\sqrt n }} = 0
\]亦即此 $\{f_n \}$ converges pointwise 到 $0$

2. 現在我們檢驗其是否為均勻收斂,由均勻收斂的 sup-norm 定義,我們可檢驗其 sup-norm 看是否收斂到 $0$;亦即檢驗
\[\left\| {{f_n} - f} \right\| = \mathop {\sup_{x \in \mathbb{R}} } \left| {\frac{{\sin nx}}{{\sqrt n }} - 0} \right| = \mathop {\sup_{x \in \mathbb{R}} } \left| {\frac{{\sin nx}}{{\sqrt n }}} \right| \le \frac{1}{{\sqrt n }}
\]故讓 $n \rightarrow \infty$可得
\[\mathop {\lim }\limits_{n \to \infty } \left\| {{f_n} - f} \right\| = 0\]

3. 此函數 sequence 導數為
\[{f_n}'\left( x \right) = \frac{1}{{\sqrt n }}n\cos nx = \sqrt n \cos nx\]

4. & 5. 此函數 sequence 的導數是否逐點收斂?
由結果 3 可知函數 sequence 的導數為 $\sqrt n \cos nx$ 此函數為在 $-1$ 與 $1$ 之間上下震盪的 cosine ,故若我們讓 $n \rightarrow \infty$ 皆不(逐點)收斂。既然此函數導數不收斂故必定不為均勻收斂。$\square$


故從上例可看出儘管原函數 sequence 具備均勻收斂,仍沒有辦法保證其導數sequence $\{ f_n'\}$依然均勻收斂。那麼問題變成 我們想知道 導數 sequence 與 原函數 sequence 之間的關係

不過我們在介紹此結果之前,我們需要一些事先工具:
===================
Mean Value Theorem 
設 $f: [a,b] \rightarrow \mathbb{R}$ 為連續函數 且 $f$ 在 $(a,b)$ 上可微。則 存在 $x \in (a,b)$ 使得
\[
|f(b) - f(a)| \le (b-a)|f'(x)|
\]===================
Proof: omitted.

====================
Theorem: Uniform Convergence Preserves Continuity
假設 $f_n \rightarrow f$ 均勻收斂在 $E \subset X$,令 $x$ 為 $E$ 上的 limit point,且假設 $\lim_{t \rightarrow x} f_n(t) = A_n(x)$ 對 $n = 1,2,3,...$則
1. $\{A_n \}$ 收斂
2. $\displaystyle \lim_{t \rightarrow x}f(t) = \displaystyle \lim_{n \rightarrow \infty} A_n(x)$
====================
Proof: omitted.


我們將此結果記做以下定理:
=======================
Theorem: Uniform Convergence and Differentiation Property
假設 $\{ f_n\}$ 為在封閉區間 $[a,b]$上可微的函數 sequence,且存在某點 $x_0 \in [a,b]$ 使得 sequence $\{ f_n(x_0)\}$ 收斂。現若 函數導數sequence $\{ f_n'\}$ 在 $[a,b]$ 上均勻收斂,則
1. 原函數sequence  $\{f_n \}$ 在 $[a,b]$上均勻收斂到某函數 $f$ 且
2. 對任意 $x \in [a,b]$,我們有
\[f'\left( x \right) = \mathop {\lim }\limits_{n \to \infty } {f_n}'\left( x \right)\]
=======================
Proof
我們先證 函數sequence  $\{f_n \}$ 在 $[a,b]$上均勻收斂到某函數 $f$ (但不知道此函數 $f$ 是否存在,我們必須現證明此函數收斂才可說 $f$ 存在),故給定 $\varepsilon >0$ 我們要證明: 存在 $N$ 使得 $n ,m> N$ 與 $x \in [a,b]$,
\[
|f_n(x) - f_m(x)| < \varepsilon \ \ \ \ (\star)
\](亦即使用 Cauchy criterion 判斷均勻收斂)。

注意到我們已知 "存在某點 $x_0 \in [a,b]$ 使得 sequence $\{ f_n(x_0)\}$ 收斂",故 $\{f_n \}$為 Cauchy,亦即存在 $N$ 使得 $n,m > N$
\[
|f_n(x_0) - f_m(x_0)| < \varepsilon \ \ \ \ (*)
\]且又知道 "函數導數sequence $\{ f_n'\}$ 在 $[a,b]$ 上均勻收斂",故我們直接取前面 Cauchy 要求的 $N$ 使得
\[
|f_n'(x) - f'(x)| < \varepsilon/(b-a)
\]觀察上式,若 $n > N$ 則我們有 $|f_n'(x) - f'(x)| < \varepsilon/(b-a)$  且由於 "  $\{ f_n\}$ 為在封閉區間 $[a,b]$上可微的函數 sequence",我們知道 $f_n - f_m$ 亦為可微函數,故對 $f_n - f_m$ 使用 Mean Value Theorem 可巧妙整合 $(*)$,亦即我們有 對任意 $x, t \in [a,b]$ , $n,m >N$ 則
\[\begin{array}{l}
|\left( {{f_n}(x) - {f_m}(x)} \right) - \left( {{f_n}(t) - {f_m}(t)} \right)| \le |x - t| \cdot |{f_n}'(x) - {f_m}'(x)|\\
 \Rightarrow |\left( {{f_n}(x) - {f_m}(x)} \right) - \left( {{f_n}(t) - {f_m}(t)} \right)| \le |x - t|\frac{\varepsilon }{{b - a}} < \varepsilon
\end{array}\]上式最後的不等式成立因為 $|x - t| \le |b -a|$。

現在我們觀察
\[\begin{array}{l}
|{f_n}(x) - {f_m}(x)| = |{f_n}(x) - {f_n}({x_0}) + {f_n}({x_0}) - {f_m}({x_0}) + {f_m}({x_0}) - {f_m}(x)|\\
\begin{array}{*{20}{c}}
{}&{}&{}&{}&{}&{}&{}
\end{array} \le \underbrace {|{f_n}(x) - {f_n}({x_0}) + {f_m}({x_0}) - {f_m}(x)|}_{ < \varepsilon } + \underbrace {|{f_n}({x_0}) - {f_m}({x_0})|}_{{\rm{ < }}\varepsilon }\\
\begin{array}{*{20}{c}}
{}&{}&{}&{}&{}&{}&{}
\end{array} < {\rm{2}}\varepsilon
\end{array}
\]故我們證明了 $\{ f_n \}$ 在 $[a,b]$上為 Cauchy (滿足 Cauchy criterion) 故 $\{f_n  \}$ 在 $[a,b]$上均勻收斂。

由於我們已知 $\{ f_n\}$ (均勻)收斂故現在可令
\[
f(x) := \lim_{n \rightarrow \infty} f_n(x), \;\; (x \in [a,b])
\]----
現在給定 $x \in [a,b]$,我們開始證明 2:亦即要證
\[f'\left( x \right) = \mathop {\lim }\limits_{n \to \infty } {f_n}'\left( x \right)
\]我們首先定義兩輔助函數 $\phi_n, \phi$ 如下:對 $t \in [a,b]$ 且 $t \neq x$,
\[{\phi _n}\left( t \right): = \frac{{{f_n}\left( t \right) - {f_n}\left( x \right)}}{{t - x}};\;\;\phi \left( t \right): = \frac{{f\left( t \right) - f\left( x \right)}}{{t - x}}
\]注意到上式 $\phi_n$,若我們讓 $n=1,2,3...$  ,由導數定義可知
\[\mathop {\lim }\limits_{t \to x} {\phi _n}\left( t \right) = {f_n}'\left( x \right) \ \ \ \ (\star)
\]我們希望輔助函數 $\phi_n$ 可以均勻收斂到 $\phi$,由於已知 $\mathop {\lim }\limits_{t \to x} {\phi _n}\left( t \right) = {f_n}'\left( x \right)$ ,故一旦  $\phi_n$ 可以均勻收斂到 $\phi$, 則透過 Theorem of Uniform Convergence Preserves Continuity 可知
1. $\{f_n' \}$ 收斂
2. $\displaystyle \lim_{t \rightarrow x}\phi (t) = \displaystyle \lim_{n \rightarrow \infty} f_n'$

故我們開始證明 $\phi_n$ 均勻收斂到 $\phi$,現在觀察
\[\begin{array}{l}
\left| {{\phi _n}\left( t \right) - {\phi _m}\left( t \right)} \right| = \left| {\frac{{{f_n}\left( t \right) - {f_n}\left( x \right)}}{{t - x}} - \frac{{{f_m}\left( t \right) - {f_m}\left( x \right)}}{{t - x}}} \right|\\
\begin{array}{*{20}{c}}
{}&{}&{}&{}&{}&{}&{}
\end{array} = \left| {\frac{1}{{t - x}}} \right|\left| {\left( {{f_n}\left( t \right) - {f_n}\left( x \right)} \right) - \left( {{f_m}\left( t \right) - {f_m}\left( x \right)} \right)} \right| \ \ \ \ (**)
\end{array}
\]再次利用 Mean Value Theorem 對 $f_n - f_m$ 且讓 $n,m > N$可得
\[\left| {\left( {{f_n}\left( t \right) - {f_m}\left( x \right)} \right) - \left( {{f_n}\left( t \right) - {f_m}\left( x \right)} \right)} \right| \le \left| {t - x} \right|\left| {{f_n}'\left( t \right) - {f_m}'\left( t \right)} \right| < \left| {t - x} \right|\frac{\varepsilon }{{b - a}}  \]
故 $(**)$ 變為
\[\left| {{\phi _n}\left( t \right) - {\phi _m}\left( t \right)} \right| < \left| {\frac{1}{{t - x}}} \right|\left| {t - x} \right|\frac{\varepsilon }{{b - a}} = \frac{\varepsilon }{{b - a}}
\]由於 $b-a$ 為有界,故我們可透過讓 $n,m$ 足夠大使得 $|\phi_n - \phi_m| \rightarrow 0$;亦即 我們證明了 $\{\phi_n \}$ 為 Cauchy ,故若 $t \neq x$,則 $\phi_n$ 均勻收斂 (到 $f_n'$);另外由於我們已知 $f_n$ 均勻收斂到 $f$,故我們可令
\[\mathop {\lim }\limits_{n \to \infty } {\phi _n}(t): = \underbrace {\phi (t)}_{ = {f_n}'\left( x \right)},\;\;t \ne x,t \in [a,b]\]
總結手邊的結果,現在我們有 $\phi_n \rightarrow \phi$ 均勻收斂,且由 $(\star)$ 可知 $\mathop {\lim }\limits_{t \to x} {\phi _n}\left( t \right) = {f_n}^\prime \left( x \right)$ ,故透過 Theorem of Uniform Convergence Preserves Continuity  可知
1. $f_n'$ 收斂
2. 且
\[\begin{array}{l}
\mathop {\lim }\limits_{t \to x} \phi (t) = \mathop {\lim }\limits_{n \to \infty } {f_n}'(x)\\
 \Rightarrow f\left( t \right) = \mathop {\lim }\limits_{n \to \infty } {f_n}'(x)
\end{array}\]

延伸閱讀

[數學分析] 逐點收斂與均勻收斂(2) - Series version

令 $X$ 為 metric space。現在考慮 一組 函數 sequence $\{f_n \}$ 定義在集合 $E \subset X$,我們稱 $\{f_n \}$為 uniform convergence 若下列任一條件成立
1. (Definition) 對任意 $\varepsilon >0$, 存在 $N >0$ 使得 $n > N$ 對所有的 $x \in E$
\[|f_n(x) - f(x)| < \varepsilon
\]2. (Cauchy criterion) 若對任意 $\varepsilon >0$, 存在 $N >0$ 使得 對任意 $x \in E$, 我們有 \[
 n,m > N \Rightarrow |f_n(x) - f_m(x)| < \varepsilon
\]3. (Sup-norm version) 若 \[
\lim_{n \rightarrow \infty} \sup_{x \in E} |f_n(x) - f(x)| =\lim_{n\rightarrow \infty}||f_n - f|| =0
\]
那麼現在我們看看若是一個級數而言,我們亦可討論此級數 是否 uniform convergence。故我們先給定級數收斂的定義

====================
Definition: (Convergence of Series of numbers )
令級數 $ \sum_{n=1}^{\infty}a_n$, 其中 $a_n \in \mathbb{R}$ ,我們稱此級數收斂若下列條件成立:
對任意 $\varepsilon >0$,存在 $N>0$ 使得 對任意 $m > N$
\[\left| {\sum\limits_{n = 1}^\infty  {{a_n}}  - \sum\limits_{n = 1}^m {{a_n}} } \right| < \varepsilon \]亦即所謂的級數的 partial sum 收斂。
====================

那麼對於一組函數級數的收斂該怎麼定義呢?

====================
Definition: (Convergence of Series of functions )
$E \subset X$,給定 $\{f_n(x) \}$ 為在 $E$ 上的函數 sequence,定義 函數級數 $ \sum_{n=1}^{\infty}f_n(x)$, 其中 $a_n \in \mathbb{R}$ ,我們稱此 函數級數為 逐點收斂(converges pointwise) 若下列條件成立:
給定任意點 $x \in E$,對任意 $\varepsilon >0$,存在 $N>0$ 使得 對任意 $m > N$
\[\left| {\sum\limits_{n = 1}^\infty  {{f_n(x)}}  - \sum\limits_{n = 1}^m {{f_n(x)}} } \right| < \varepsilon \]亦即所謂的級數的 partial sum 收斂。

我們稱此 函數級數 為 均勻收斂(converges uniformly) 若下列條件成立:
對任意 $\varepsilon >0$,存在 $N>0$ 使得 對任意 $m > N$ 與 任意 $x \in E$
\[\left| {\sum\limits_{n = 1}^\infty  {{f_n(x)}}  - \sum\limits_{n = 1}^m {{f_n(x)}} } \right| < \varepsilon \]亦即所謂的級數的 partial sum 收斂。
====================

注意到上述定義要求無窮級數合,這在使用上會有一定的困難,故我們轉而利用 Cauchy criterion  (因為 "級數" 本身可視為一個函數。則透過函數的均勻收斂條件可知道 若且唯若 Cauchy criterion 成立 ),亦即:

Cauchy Criterion for uniform convergent series
函數級數 $ \sum_{n=1}^{\infty}f_n(x)$ 為 均勻收斂(converges uniformly)若下列條件成立
對任意 $\varepsilon >0$, 存在 $N >0$ 使得 $n,m > N (n>m>N)$, $x \in E$ 我們有\[\left| {\sum\limits_{k = 1}^n {{f_k}(x)}  - \sum\limits_{k = 1}^m {{f_k}(x)} } \right| < \varepsilon \]


接著我們在給出一個更進一步的結果,就是若函數級數的每一項都有界,且其有界函數的級數合收斂,則原函數級數有 均勻收斂,此稱作 Weierstrass M test 記做下面定理
====================
Theorem: Weierstrass M-test
設 $\{f_n \}$ 為一組定義在集合 $E$ 上函數 sequence,且我們假設
\[
|f_n(x)| \le M_n, (x\in E, n\in \mathbb{N})
\] 現若 級數 $\sum_n M_n$ 收斂,則 $\sum_n f_n$ converges uniformly on $E$ 。
====================

Proof:
給定任意 $\varepsilon >0$,我們可使用 Cauchy criterion 幫助我們判斷 函數級數的 均勻收斂;現在觀察 partial sum:
\[\left| {\sum\limits_{k = 1}^n {{f_k}(x)}  - \sum\limits_{k = 1}^m {{f_k}(x)} } \right| = \left| {\sum\limits_{k = m}^n {{f_k}(x)} } \right| \le \sum\limits_{k = m}^n {\left| {{f_k}(x)} \right|}  \le \sum\limits_{k = m}^n {{M_k}} \ \ \ \ (*)
\]上述不等式最後一項成立 (使用假設 $|f_k(x)| \le M_k, (x\in E, k\in \mathbb{N})$),又因為 級數 $\sum_n M_n$ 收斂,故可知 $(*)$ 收斂,亦即存在一個 $N$ 使得 當 $m,n > N$ 的時候,
\[\left| {\sum\limits_{k = 1}^n {{f_k}(x)}  - \sum\limits_{k = 1}^m {{f_k}(x)} } \right| \le \sum\limits_{k = m}^n {{M_k}}  < \varepsilon \]亦即 $\sum_n f_n$ converges uniformly on $E$

Example
考慮級數
\[
\sum_{n=1}^{\infty} \frac{\sin (nx)}{n^2}
\]試判斷此級數是否均勻收斂?
Solution
利用 Weierstrass M-test,首先檢驗
\[
 |\frac{\sin (nx)}{n^2}| \le |\frac{1}{n^2}|
\]接著觀察
\[
\sum_{n=1}^\infty \frac{1}{n^2} \rightarrow 0
\]故 由Weierstrass M-test Theorem
\[
\sum_{n=1}^{\infty} \frac{\sin (nx)}{n^2} \rightarrow 0
\]均勻收斂。


延伸閱讀
[數學分析] 逐點收斂與均勻收斂(3) - Differentiation property

5/11/2011

[衍生商品] 希臘值與動態避險 (2) - Gamma and Gamma Neutrality


延續上篇 [衍生商品] 希臘值與動態避險 (1)-Delta Hedging Example ,這次要介紹 希臘值 Gamma: $\Gamma$,此參數定義為
\[
\Gamma := \frac{\partial^2 f}{\partial S^2}
\] 亦即為標的資產價格 $S$ 的二次偏導數。

注意到之前我們定義過 $\Delta := \frac{\partial f}{\partial S}$,故 $\Gamma$ 可視為選擇權 $\Delta $ 的變化 與 標的資產價格 $S$ 變化的比率。

Comment
1. 當 $\Gamma $ 很小的時候,表示 $\Delta$ 變化緩慢 (stable $\Delta$) (亦即對標的資產價格變動不敏感),故此時對於 $\Delta$-Hedging 所需的 Rebalance 不需太過頻繁。但是若 $\Gamma$ 很大的時候,表示 $\Delta$ 變化劇烈,亦即對標的資產價格變動非常敏感,故此時 $\Delta$-Hedging 需要頻繁的做 Rebalance 來確保 Delta-Neutral ($\Delta =0$)。

2. 如果考慮的是一個 選擇權交易組合的 $\Gamma$,則其定義為
\[
\Gamma := \frac{\partial^2 \Pi}{\partial S^2}
\] 其中 $\Pi$ 為選擇權交易投資組合的價格。

假定且我們假設 標的資產的波動度為 Constant,則投資組合的價格為資產價格 $S$ 與 時間 $t$ 的函數,亦即我們可對 $\Delta \Pi$ 做泰勒展開求資產價格的變化
\[
\small{\Delta \Pi  = \underbrace {\frac{{\partial \Pi }}{{\partial S}}}_\Delta \Delta S + \underbrace {\frac{{\partial \Pi }}{{\partial t}}}_\Theta \Delta t + \frac{1}{2}\underbrace {\frac{{{\partial ^2}\Pi }}{{\partial {S^2}}}}_\Gamma \Delta {S^2} + \frac{1}{2}\frac{{{\partial ^2}\Pi }}{{\partial {t^2}}}\Delta {t^2} + \frac{1}{2}\frac{{{\partial ^2}\Pi }}{{\partial S\partial t}}\Delta S\Delta t + ...
}\]其中 $\Delta \Pi$ 與 $\Delta S$ 為 很小的時間區間 $\Delta t$ 內投資組合的價格與股票價格的變化。

現在如果此 投資組合為 Delta-Neutral,亦即 $\Delta=0$,且我們忽略高階項的效果,則上述泰勒展開可改寫為
\[\Delta \Pi  \approx \Theta \Delta t + \frac{1}{2}\Gamma \Delta {S^2} \ \ \ \ (*)
\]
下圖展示了 Delta Neutral 的投資組合 $\Delta \Pi$ 與 $\Delta S$的關係。


上圖顯示了如果是 Long option (則此時標的股價上升則投資組合價格跟著上升),則 $\Gamma >0, \Theta <0$
如果是 Short option ,則 $\Gamma <0, \Theta >0$

3. 對於 European Call option 與 European Put option,B-S formula 可直接求得 $\Gamma$ 的解析式如下:
\[
\Gamma  = \frac{{N'\left( {{d_1}} \right){e^{ - qT}}}}{{{S_0}\sigma \sqrt T }}
\] 其中 $N'(\cdot)$ 為 Standard Normal density function。
\[N'\left( {{d_1}} \right) = \frac{1}{{\sqrt {2\pi } }}{e^{\frac{{ - d_1^2}}{2}}}\]

現在我們看個例子:

=============================

Example (Gamma Neutrality)
假定某標的資產投資組合為 Delta Neutral,其對應的 Gamma 值為 $-10000$,現在假設標的資產價格 $\Delta S$ 在短時間內變化為 $+2$ 或者 $-2$ (假設短時間 $\Delta t \approx 0$),則交易組合的價值變動為何?

============================

Solution
由於此投資組合已經為 Delta Neutral,故 $\Delta =0$,我們可利用之前推導的結果: $(*)$
\[\begin{array}{l}
\Delta \Pi  \approx \Theta \Delta t + \frac{1}{2}\Gamma \Delta {S^2}\\
 \Rightarrow \Delta \Pi  \approx 0 + \frac{1}{2}\left( { - 10000} \right){\left( 2 \right)^2} = -20000
\end{array}\]亦即交易組合價值下跌 $20000$ 元。 $\square$

再者我們介紹如何使投資組合進一步具備 Gamma Neutral 的特性

Gamma Hedging and Gamma Neutral
假定一個 Delta Neutral 的投資組合的Gamma 值為 $\Gamma$,而某交易所的交易選擇權的Gamma 值為 $\Gamma_T$,現若決定將 $w_T$ 數量的選擇權加入到原本的投資組合中,則新的投資組合的 Gamma 值為 $\Gamma^*$
\[
\Gamma^* =\Gamma + w_T \Gamma_T
\]故如果我們要使投資組合為 Gamma-Neutral,亦即 $ \Gamma^* =0$則我們需用
\[\begin{array}{l}
{\Gamma ^*} = \Gamma  + {w_T}{\Gamma _T} = 0\\
 \Rightarrow {w_T} =  - \frac{\Gamma }{{{\Gamma _T}}}
\end{array}
\] 亦即須增加/減少 $w_T$ 個選擇權,但注意到當我們達成 Gamma Neutral 的時候,很可能會變動到原本的 Delta,故我們需回頭調整 Delta 來保證薪投資組合除了 Gamma Netural 之外亦維持 Delta Netural。

以下我們看個例子如何同時達成 Gamma 與 Delta Neutral。

Example: Delta & Gamma Neutrality
考慮一投資組合為 Delta Neutral,且其 $\Gamma = -3000$,而對應於交易所交易的選擇權的 $\Delta = 0.62, \Gamma=1.50$,試建立一個新的 投資組合使得其 Gamma 與 Delta Neutral

Solution
STEP1 : 首先對付 Gamma
令 $w_T$ 為需交易的選擇權數量。則我們要 $\Gamma^*=\Gamma  + {w_T}{\Gamma _T} = 0$故
\[
-3000 + w_T 1.50 = 0\\
\Rightarrow {\rm{ }}{w_T}{\rm{ }} = 2000
\] 故需買入 2000 份選擇權來達成 Gamma Neutral。

STEP2: 找出 Gamma Hedging 之後的 Delta 為多少?
再者由於我們對原本 Delta Neutral 的投資組合中 加入了 2000份的選擇權,故 Delta 被更動,不再是 Delta Neutral。新的 Delta 為
\[
2000 \times 0.62 = 1240
\]亦即透過 Gamma Hedging 之後我們新的投資組合多了 $1240 $ 的 Delta

STEP3 : 對付 Delta :
由於我們多了 1240 的 Delta ,故需賣出1240股達成 $\Delta$ hedging。
(注意到賣出標的股票並不會影響 $\Gamma$ (WHY?! 因為 $\frac{\partial^2}{\partial S^2} S =0$ ) $\square$


接著我們看個稍微複雜一點的例子:

Example (Delta/Gamma Neutral via Put and Call combination )
假定某公司股價最近經歷一連串的下跌,投資者預期該公司股價將持續下跌,故決定購買 $100$ 份  執行價格為 $20$,到期時間為三個月,價格 為 0.75 元的 European Put option 來獲利。現考慮市場條件為 無風險利率 $4 \%$,當前股價為 $22$,股價波動度為 $39 \%$,且該公司不配發股息。

(a) 如果當前股價突然上漲到 $25$ 元/ 每股,則投資人所購入的 Put option 策略 賺/賠 多少?

(b) 現在假設投資者採用 Delta Hedging (在股價原本為 $22$ 的時候) 來保護其購買的 Put option 策略,那麼其淨利為何?

(c) 假設投資人決定再使用執行價格為 $20$ 到期時間為三個月的 European Call option 來達成 Delta-Gamma Neutral  (在股價原本為 $22$ 的時候)。試求投資人應如何達成此策略? 其淨利為何?

Solution (a)
改寫已知資訊如下:
\[
K=20, T=3/12, P_{S=0.22}=0.75, r=0.04, S_0=22, \sigma=0.39, q=0
\] 由於股價上漲到 $25$,故連帶的 Put option 價格也會有所變動。
透過 B-S formula 計算 股價上漲到 $25$ 元後的 Put option 價格
首先計算
\[\left\{ {\begin{array}{*{20}{l}}
{{d_1} = \frac{{\ln \left( {{S_0}/K} \right) + \left( {r - q + {\sigma ^2}/2} \right)T}}{{\sigma \sqrt T }} = 1.2931}\\
{{d_2} = {d_1} - \sigma \sqrt T {\rm{ = 1}}{\rm{.0981}}}
\end{array}} \right.\]
帶入 B-S formula:
\[\begin{array}{l}
P = K{e^{ - rT}}N\left( { - {d_2}} \right) - S{e^{ - qT}}N\left( { - {d_1}} \right)\\
 \Rightarrow P = 20{e^{ - 0.04 \times \frac{3}{{12}}}}N\left( { - 1.0981} \right) - 25N\left( { - 1.2931} \right) = 0.2488
\end{array}
\]亦即股價上漲後的 Put option price 為 $P_{S=25} = 0.2448$ 元

故可知投資人的 Put option 投資策略 淨利為
\[ (0.2448-0.75)100=-50.52 \]

Solution (b)
現在由於投資人要進行 Delta-Hedging,故我們首先必須求得 Delta 值,由題意可知我們需計算上漲前的 Delta: $\Delta_{S=22, put}$
又由於 Put option 的 Delta 為 $\Delta_{S=22, put}=-e^{-qT} N(-d_1)$,故我們計算 $d_1$ (注意! 此時的 $d_1$ 為股價上漲前的,不可直接使用 part (a) 所計算出來的結果)
\[{d_1} = \frac{{\ln \left( {22/20} \right) + \left( {0.04 - 0 + \frac{{{{\left( {0.39} \right)}^2}}}{2}} \right)\frac{3}{{12}}}}{{0.39\sqrt {3/12} }} = 0.6376
\] 故
\[
\Delta_{S=22, put}=-e^{-qT} N(-d_1) = -0.2619
\] 又由於投資人購買了 100份 Put option,故此投資組合的總 $\Delta$ 為
\[
\Delta = -0.2619 \times 100 = -26.19
\] 所以若欲獲得 Delta-Neutralality,則必須 購入 26.19 股股票。


Solution (c)
現在由於投資人要進行 Delta-Gamma Hedging,故我們首先必須先對付 Gamma:
也就是要先求得 Gamma 值,由題意可知我們需計算上漲前的 Gamma :
\[\Gamma  = \frac{{N'\left( {{d_1}} \right){e^{ - qT}}}}{{{S_0}\sigma \sqrt T }}\]
其中
\[N'\left( {{d_1}} \right) = \frac{1}{{\sqrt {2\pi } }}{e^{\frac{{ - d_1^2}}{2}}} = \frac{1}{{\sqrt {2\pi } }}{e^{\frac{{ - \left( {0.6379} \right)_{}^2}}{2}}} = 0.3255\]
將上式代入 Gamma 可得
\[\Gamma  = \frac{{N'\left( {{d_1}} \right){e^{ - qT}}}}{{{S_0}\sigma \sqrt T }} = \frac{{0.3255}}{{22\left( {0.39} \right)\sqrt {3/12} }} = 0.0759
\]注意到上式的 Gamma for put $=$ Gamma for call;亦即 $\Gamma_{call} = \Gamma_{put}$

現在我們考慮加入 $w_T$ 份 Call option,其對應的 $\Gamma_{call} = \Gamma_{put} = 0.0759$,故總 Gamma 為 原本購買 100 份 Put option 的 Gamma 加上 $w_T$ 份 Call option 的 Gamma;又因為我們要達成 Gamma Neutral,故總 Gamma 必須為零;故我們可求解到底需要多少份 call option:
\[\begin{array}{l}
\Gamma  = 0.0759 \times 100 + {w_T}0.0759 = 0\\
 \Rightarrow {w_T} =  - 100
\end{array}
\] 亦即須要賣出 100 份 Call option 即可達成 Gamma Neutral。

但由於我們更動了 Gamma,故 Delta 亦會連帶受到更動;其更動後 (賣出 100 份 Call ) 的 Delta 為
\[
\Delta = -100 e^{-qT} N(d_1) = -100 N(0.6379) = -73.82
\] 故我們需要購入 73.82 股股票 ,即可達成 Delta-Neutral。

總結 Delta-Gamma Hedging 策略 如下:
買入 100 份 Put option (原始策略)
購入 26.19 股股票 (Delta-Neutral)

賣出 100 份 Call option (透過 Call option 達成 Gamma Neutral)
購入 73.82 份 股票 (修正 Gamma Neutral所造成的 Delta 變動)

故總計為
買入 100份 Put
賣出 100份 Call
購入 26.19+73.82=100 股股票

其淨利為: (股價為 $22$ 時 Call option 價格為 $2.9530$;股價上漲到 $25$ 元的 Call option 價格為 5.4438 (此兩數值可透過 B-S model 計算而得或者 Put-call parity))
\[\underbrace {(25 - 22) \times 100}_{Long\begin{array}{*{20}{c}}
{}
\end{array}stock} + \underbrace {\left( {0.2448 - 0.75} \right) \times 100}_{Long\begin{array}{*{20}{c}}
{}
\end{array}put} - \underbrace {\left( {5.4438 - 2.9530} \right) \times 100}_{Short\begin{array}{*{20}{c}}
{}
\end{array}Call} = 0.4\]




ref: John C. Hull, Options, Futures and Other Derivatives 7th.

[衍生商品] 希臘值與動態避險 (1)-Delta Hedging Example

回憶前篇 [衍生商品] 希臘值與動態避險 (0) - Delta and Delta Neutral ,這次要介紹如何利用 $\Delta$ 進行動態避險。

回憶 $\Delta$ 定義如下:
\[
\Delta := \frac{\partial f}{ \partial S}
\]亦即表示為 選擇權價格 $f$ 對 股價 $S$ 的變化率。(由於其為一階導數,故為斜率)

現在來看個例子:

Example 1 : (Delta Hedging)

如果 $\Delta = 0.6$ 則表示當股價 些微變化 的時候,對應的選擇權價格變化大約是股價變化值 的 $60 \%$。
下圖顯示了一組 $\Delta$ 值在某時刻的例子:


考慮上圖,假設股價為 $\$ 100 $,Call option 價格為 $ c= \$ 10$,現在考慮某金融機構的交易員賣出了 $20$ 份 Call option (一份選擇權對應其持有者可以有權購買 $100$ 股,亦即 $20$ 份call option 共 $x= 20 \times 100 = 2000$ 股)。此時如果不進行避險,則當股價上升時,該交易員會暴露風險之中:

簡單的說,現在有兩個人物:
  1. 賣出 call option 的交易員
  2. 跟交易員 購買 call option 的客戶
此時客戶的 $\Delta_{Customer} =0.6$ (因為購入call option,當股價上升對顧客有利,此時 $\Delta >0$)
而交易員的 $\Delta_{trader} = -0.6$, (由於交易員是 "賣出" 選擇權,故當股票價格上升,則選擇權會被執行,此情形時將對交易員產生風險。故此 $\Delta$ 對 交易員而言是負值)

現在,站在交易員的觀點,如果不進行避險,則交易員本身的潛在損失為
\[
-0.6 \times 2000 =-1200 \ \text{shares}
\]
我們必須消除賣出 Call option所帶來的 風險,此時交易員可進行 $\Delta$-Hedging  來補足缺少的 $1200$ 股 股票。:

由於交易員是 "賣出" Call option ,故避險方法便是進行反向操作,也就是可以透過 "買入" 一定量股票來抵銷當 股價上升時,Call option 被執行所帶來的損失風險,故
購買 $\Delta \times x = 0.6 \times 2000 =1200$ 股股票

此時如果 股票上漲 $1$ 元,則交易員 買入的股票會上升 $1200$ 元 (賺 $1200$ 元),而由 圖中 $\Delta =0.6$ 可知 Call option 會上漲 $0.6$ 元,故如果此時 Call option 被持有者執行,則交易員會損失 $0.6 \times 2000 = \$ 1200$ 此數值剛好會跟交易員進行避險時候買入的股票所賺取的 $1200$ 抵銷。

相反的如果股票下跌 $1$元,則交易員 買入的股票會下跌 $1200$ 元 (損失 $1200$ 元),而由圖中 $\Delta =0.6$ 可知 Call option 亦會下跌 $0.6$ 元,此時選擇權不會被執行,則交易員因為賣出選擇權 會賺得 $ \$ 1200$ 此數值剛好會跟交易員進行避險時候賣出的股票所損失的 $1200$ 抵銷。 $\square$

Comment:
1. 注意到上述例子中,由於 $\Delta$ 會變動,故抵銷後的 $\Delta$-Hedging 只能維持一段極短時間,也就是需要不斷的調整 $\Delta$-Hedging ,此稱為 Rebalancing。一般而言,隨時間不斷調整的 避險策略 統稱為 $Dynamic Hedging$,這邊展示的是利用 $\Delta$ 進行避險

2. $\Delta$ 在 Nondividend Black-Scholes formula 中等價為 $N(d_1)$;亦即
\[
\Delta := \frac{{\partial f}}{{\partial S}} = N\left( {{d_1}} \right)
\] 其中 $N(\cdot)$ 為 Cumulative normal distribution。

3. $\Delta$ -Hedging 並非 Perfect Hedging。(WHY!? 理由同 comment 1 )

Example 2 (Delta-Hedging )
考慮 造市商(market-maker) 賣出 $K=40$ 的 call-option on 100 股 股票,且
$ \sigma=0.3$, $r=0.08 $ 連續複利
現在考慮 Day 0,$S=\$40$, $c =\$2.78$, $\Delta=0.58$
如何進行 $\Delta$-hedging?
How much cost you to create such a $\Delta$-hedging ?

現在再考慮 Day 1,$S=40.5, c= \$ 3.06$, $\Delta=0.61$
如何進行 $\Delta$-hedging?
Overnight Mark-to-market profit/loss ?

現在再考慮 Day 2,$S=39.25, c= \$2.328$, $\Delta=0.53$
Overnight Mark-to-market profit/loss ?

Solution:
考慮 時刻為 DAY 0
首先考慮不進行避險情況,market-maker 本身為 $-\Delta \times 100 = -58$,故需要補足此 $58$ 股股票,亦即需要購買 $58$ 股 股票即可達成 DAY0 $\Delta$-Hedging。

接著我們可以計算要花多少錢才可以建構此避險策略: ( 賣出選擇權 與 購入 股票之後的花費):
\[
58 \times 40 - 2.78 \times 100 = \$ 2042
\]故我們知道建構此避險策略需要花費 $\$ 2042$,故我們可借入此金額並考慮利率,亦即我們借入 $2042 \times e^{8\%/365} = 2042.45$,故利息為 $2042.45-2042=\$ 0.45$。現在總結 Day 0 如下:

購入 $58$ 股股票 達成避險,然後我們需要借入 $2042$  元 並支付利息 $0.45$ 來達成此避險策略。

接著考慮 DAY 1
此時相關資訊(股價、選擇權價格、Delta)變動為 $S=40.5, c= \$ 3.06$, $\Delta=0.61$

我們可以先行計算 Overnight mark-market profits/loss :
\[
(40.5-40) \cdot (58) - (3.06-2.78) \cdot (100) - 0.45 = 0.55
\] 由於此時 $\Delta =0.61$ 已經改變,故我們需要重新調整股票數來消除風險。
首先考慮不進行避險情況,market-maker 本身為 $-\Delta \times 100 = -61$,故需要補足此 $61$ 股股票,但由於在 DAY0 已經購入 $58$ 股,故我們只需再購買 $61-58 = 3$ 股 股票即可達成 DAY1 $\Delta$-Hedging。

現在我們來計算達成此避險策略所需的花費 (DAY1 total cost ):
\[
61 \times 40.5 - 3.06 \times 100 = 2164.5
\] 同樣地,我們可以得知需要借入 $2164.5$ 元 且須支付利息為 $2164.5e^{8\%/365} - 2164.5=0.47$

考慮 DAY2 Mark-market Profits/loss:
此時相關資訊(股價、選擇權價格、Delta)變動為 $S=39.25, c= \$2.328$, $\Delta=0.53$
\[
 (39.25-40.5) \cdot 61 - (2.328-3.06) \cdot 100 - 0.47 = -3.52 \ \ \ \ \square
\]


ref: John C. Hull, Options, Futures and Other Derivatives 7th.

5/10/2011

[衍生商品] 淺談選擇權 (2) - Put-Call Parity

這次要介紹的是選擇權訂價一個重要的關係:買權賣權等價關係 (Put-Call Parity):

想法: 利用 Option 建構一個 合成的 Forward contract ,再來比較其 payoff

現在考慮 當前股價為 $S_0$ 的 無配發股息的股票 (Non-dividend paying stock),且 $C, P$ 為對應的 Call option 與 Put option 的價格;現在我們透過 Buying a call + Short a put 可以得到 Synthetic forward 如下圖 Payoff (點圖放大)

注意到上圖右方合成之後的 Payoff 圖等價 Long a forward ,故我們稱此利用 Option 所合成出來的 Forward 為 Synthetic forward。

另外,如果我們觀察對於 Synthetic Forward 而言,我們是透過 Buying a call + short a put 達成,故其當前的 Payoff  (Payoff Today) 可寫為 $-C + P$ (買一份 call $ = -C$,賣出一份 put $=+p$);且 Payoff at Expriation date 為 $S_T - K$

另外對於標準 Forward 而言,其 Payoff Today $=0$ (由於 Forward 並不需支付premium,故 Payoff today =0);而 Pay off at Expriation date 為 $S_T -F_0$

因為我們的 Synthetic Forward 是用來 mimic 標準的 Forward ,故此兩者之 Payoff 必須等價,我們將所有的 payoff 都折現到 Today 來比較:

Payoff Today :
\[
-C + P + PV(S_T - K) = 0 + PV(S_T - F_0) \\
\Rightarrow C-P =  PV( F_0 - K)
\] 又由於對 Forward 而言 $PV(F_0) := S_0$,故我們得到
\[
C-P =  S_0 - PV( K)
\] 上式稱為 歐式選擇權 支付股息 的 Put-Call Parity。

注意到如果上式出現
\[
C-P \neq  S_0 - PV( K)
\] 則此時出現套利機會,因為 Put-Call parity 不再成立。亦即如果出現
$C > P + S_0 - PV( K) $ 則 表示 當前 的 Call option 價格過高,可以 Short call + buy $(P+S_0 -PV(K))$ 達成套利。

或者 $C < P + S_0 - PV( K)$ 則表示當前的 call option 價格過低,可以 long call + sell $(P+S_0 -PV(K))$ 達成套利。


現在我們再問個問題,如果現在考慮 Divdend paying Stocks ,Put-Call Parity 該如何修正呢?
首先寫下一般 Put-Call Parity
\[
C-P =  PV( F_0 - K)
\] 回憶對於有 Dividend paying 的 Forward ,我們有 $PV(F_0) := S_0 - PV(D)$,故我們將上式改寫
\[
C-P =  S_0  - PV(D) -PV( K)
\] 上式即稱為歐式選擇權 支付股息 的 Put-Call Parity

如果考慮股息為連續複利 $q$,則我們
\[
C-P =  S_0 e^{-qT} -PV( K)
\]

現在我們看個例子:

Example: Put Call Parity
考慮一執行價格 $\$ 30$ 且六個月後到期的 股票歐式 Call option 其價格為 $ \$ 2$,其標的股價為 $\$ 29$,且股息為 $\$ 0.5$ 預計於二個月與五個月的時候發放,無風險利率為 $10 \%$連續複利計,試求相同規格的 Put option 合理價格 (無套利機會價格) 應為多少?

Solution
改寫已知資訊如下:
\[
K=30, C=2, S_0=29, D=0.5, r=0.1, T=6/12
\]由 Put-Call parity 可知
\[\begin{array}{l}
C - P = {S_0} - PV\left( D \right) - K{e^{ - rT}}\\
 \Rightarrow 2 - P = 29 - \left( {0.5{e^{ - 0.1 \times \frac{2}{{12}}}} + 0.5{e^{ - 0.1 \times \frac{5}{{12}}}}} \right) - 30{e^{ - 0.1 \times \frac{6}{{12}}}}\\
 \Rightarrow P = 2.51
\end{array}\]


Example: No-Arbitrage Opportunity via Put-Call Parity
考慮一執行價格為 $50$ 且到期時間為 12個月 的股票歐式 Call option ;且其價格為 $\$ 6$。同樣規格的的 股票歐式 Put Option 價格為 $\$ 5$;另外市場當前股價為 $54$,配發股息預計於六個月後配發 $4$元,連續複利無風險年利率為 $5 \%$。

試問是否存在 套利機會?

Solution
先求無套利機會價格,由 Put-Call parity
\[\begin{array}{l}
C - P = {S_0} - PV(D) - PV(K)\\
 \Rightarrow C = P + {S_0} - PV(D) - PV(K)\\
 \Rightarrow C = 5 + 54 - 4{e^{ - 0.05 \times \frac{6}{{12}}}} - 50{e^{ - 0.05 \times \frac{{12}}{{12}}}}\\
 \Rightarrow C = 7.5373
\end{array}
\]注意到由 Put-call parity 得到的結果顯示 $C = 7.5373$ 但當前的 Call price 為 $6$ 故存在套利機會:由於當前 Call price 低於 7.5373,故我們可 Long Call option 接著賣出 其餘組合如下表

\[\small{\begin{array}{l}
6 < C = P + {S_0} - PV(D) - PV(K)\\
\begin{array}{*{20}{c}}
{}&{Today}&{6\begin{array}{*{20}{c}}
{}
\end{array}month}&{At\begin{array}{*{20}{c}}
{}
\end{array}Exp.\left( {{S_T} \le K} \right)}&{At\begin{array}{*{20}{c}}
{}
\end{array}Exp.\left( {{S_T} > K} \right)}\\
\hline
{Long\begin{array}{*{20}{c}}
{}
\end{array}Call}&{ - C}&0&0&{{S_T} - K}\\
{Short\begin{array}{*{20}{c}}
{}
\end{array}Put}&{ + P}&0&{ - \left( {K - {S_T}} \right)}&0\\
{Short\begin{array}{*{20}{c}}
{}
\end{array}Stock}&{ + {S_0}}&{ - D}&{ - {S_T}}&{ - {S_T}}\\
{invest\begin{array}{*{20}{c}}
{}
\end{array}\$ D}&{ + PV\left( D \right)}&{ + D}&0&0\\
{invest\begin{array}{*{20}{c}}
{}
\end{array}\$ K}&{ - PV\left( K \right)}&0&K&K\\
{Total\begin{array}{*{20}{c}}
{}
\end{array}Gain}&{}&0&0&0
\end{array}\\
\\
 \Rightarrow \begin{array}{*{20}{c}}
{}&{Today}&{6\begin{array}{*{20}{c}}
{}
\end{array}month}&{At\begin{array}{*{20}{c}}
{}
\end{array}Exp.\left( {{S_T} \le 50} \right)}&{At\begin{array}{*{20}{c}}
{}
\end{array}Exp.\left( {{S_T} > 50} \right)}\\
\hline
{Long\begin{array}{*{20}{c}}
{}
\end{array}Call}&{ - 6}&0&0&{{S_T} - 50}\\
{Short\begin{array}{*{20}{c}}
{}
\end{array}Put}&{ + 5}&0&{ - \left( {50 - {S_T}} \right)}&0\\
{Short\begin{array}{*{20}{c}}
{}
\end{array}Stock}&{ + {54}}&{ - 4}&{ - {S_T}}&{ - {S_T}}\\
{invest\begin{array}{*{20}{c}}
{}
\end{array}\$ D}&{ - 4{e^{ - 0.05 \times \frac{6}{{12}}}}}&{ + 4}&0&0\\
{invest\begin{array}{*{20}{c}}
{}
\end{array}\$ K}&{ - 50{e^{ - 0.05 \times \frac{{12}}{{12}}}}}&0&{50}&{50}\\
{Total\begin{array}{*{20}{c}}
{}
\end{array}Gain}&{1.5373}&0&0&0
\end{array}
\end{array}}\]




[衍生商品] 淺談選擇權 (1) - Some Properties of Option

延續上篇 [衍生商品] 淺談選擇權 (0) - Moneyness and profit/payoff of Option ,我們目標是要找出合理的定價。但目前對上述並無頭緒,只知道 選擇權價值 = 內在價值與時間價值。
\[
\text{Option Value $=$ Intrinsic Value $+$ Time Value}
\] 然後 由報價中,我們可以發現有一些參數似乎會影響我們對選擇權的定價。

現在我們總結需要的參數:

$K$: 執行價格 (Strike Price)
$S_0$: 當前標的資產價格 (這邊我們以當前股價表示) (Current Stock Price)
$\sigma$: 股價波動度 (Volatility)
$T$ : 到期時間 (Expiration time)
$r$ : 無風險利率 (risk-free interest rate)
$D$: 股息 (Dividend)
Style: 美式選擇權 或者 歐式選擇權。

接著我們討論當上述參數變動的時候,會對選擇權價格造成甚麼影響?

Varying Strike Price $K$:
現在考慮兩個不同的執行價格 $K_1, K_2$ 且 $K_1 < K_2$,則我們知道對於 Call option 而言,越低的執行價格代表越此 Call option 獲利機會相對較大,故Call option 售價在較低的 執行價格 應越高
\[ C(K_1) > C(K_2) \] 對 Put Option 而言,情況則相反
\[ P(K_1) < P(K_2) \] 那麼現在如果我們考慮兩選擇權除了 Strike price 以外其餘參數皆相同,則我們有如下重要結果:
\[\left\{ \begin{array}{l} {K_2} - {K_1} \ge C({K_1}) - C({K_2}) \ge 0\\ {K_2} - {K_1} \ge P({K_2}) - P({K_1}) \ge 0 \end{array} \right.\]

Varying  Expiration Time $T$:
1. 對於美式選擇權而言,越長的 $T$ 表示有越多機會可以 執行,故 $T$ 增大 $\Rightarrow$ 選擇權價格上升
2. 如果對歐式選擇權,到期時間的效果無法看出確切關係 (越長的 $T$ 並無法保證選擇權價格上升/下降)

選擇權價格的上/下界:
對於 Call Option 而言,其 Call option price 的上下界如下圖所示:,

對於 Put Option 而言,其 Put option price 的上下界如下圖所示:
現在我們看個例子:

Example: (Lower Bound of call option)
現在考慮一個 6個月到期且不支付股息的 Call option,當前股價為 $\$ 80$,執行價格為 $\$75$,且無風險年利率為 $10 \%$ 以連續複利計。試求其選擇權的下界應為何?

Solution
由於此為 Call option,我們知道其下界為
\[\begin{array}{l}
\max \{ {S_0} - PV(K),0\} \\
 \Rightarrow \max \{ 80 - 75{e^{10\%  \times \left( {6/12} \right)}},0\}  = \max \{ 8.66,0\}  = 8.66
\end{array}\]


延伸閱讀
[衍生商品] 淺談選擇權 (2) - Put-Call Parity

[衍生商品] 淺談選擇權 (0) - Moneyness and profit/payoff of Option

選擇權 (Option) 定義: 為一個非綁定合約,給予 持有者 在合約上約定的 (未來)到期日期 (Expriation date),依照合約上約定的 執行價格 (Strike price),買入或者賣出標的資產 (EX: 股價、指數、外匯利率) 的權利。

Comment
1. 上述 非綁定合約,意指 持有者不一定要履行選擇權。亦可選擇放棄履行
2. 選擇權 與 期貨 (futures) 最大差別在於 選擇權買方需先支付權利金 (premium) 給賣方。但期貨無須先支付權利金。

選擇權依照分類有兩種

1. 買權 (Call option): 
在指定到期日期,持有者有權利以 Strike price 購買 標的資產
下圖為購買一組買權: Long (=Buy) a Call Option 在到期日的 Payoff 與 Profits (點圖放大)


上圖中 $C$ 表示 Call option 價格,而 $FV(C)$ 則為 到期時刻 Call option 的 future value,$K$ 為執行價格, $S_T$ 為到期日時的股價。其 Payoff 與 Profit 用數學表示可寫為:
\[\left\{ \begin{array}{l}
{\rm{Payoff}}{{\rm{}}_{Call}}:\max \left\{ {{S_T} - K,0} \right\}\\
{\rm{Profit}}{{\rm{}}_{Call}}:\max \left\{ {{S_T} - K,0} \right\} - FV\left( C \right)
\end{array} \right.\]

2. 賣權 (Put option): 
在指定到期日期,持有者有權利以 Strike price 賣出 標的資產
下圖為 購買一組賣權 Long (=Buy) a Put Option 在到期日的 Payoff 與 Profits (點圖放大)
上圖中 $P$ 表示 Put option 價格,而 $FV(P)$ 則為 到期時刻 Put option 的 future value,$K$ 為執行價格, $S_T$ 為到期日時的股價。其 Payoff 與 Profit 用數學表示可寫為:
\[\left\{ \begin{array}{l}
{\rm{Payoff}}{{\rm{}}_{Put}}:\max \left\{ {K - {S_T},0} \right\}\\
{\rm{Profit}}{{\rm{}}_{Put}}:\max \left\{ {K - {S_T},0} \right\} - FV\left( P \right)
\end{array} \right.\]


另外亦可依照 何時可以執行 來分類
  1. 歐式選擇權 (European Option) : 只能在到期日才能執行 買/賣權。
  2. 美式選擇權 (American Option) : 在到期日之前都能執行 買/賣權。
Comment:
 可以想見由於 美式選擇權給予持有者更大的彈性決定甚麼時候要執行,故此選擇權價格必定會高於 歐式選擇權。亦即如果我們稱 $C$ 為 Call option 價格, $P$ 為 Put Option 價格,則 美式與歐式之間有如下關係:
\[
C_{American} \geq C_{European}\\
P_{American} \geq P_{European}
\]另外今日大多數選擇權交易都以美式選擇權為主。


有了上述觀念,我們可以在介紹新的專有名詞,稱作 Moneyness of Option:

術語:Moneyness of Option 
一般而言,選擇權立刻執行的當下,我們可針對其 Payoff  (立刻執行的當下,選擇權的價格) 進行討論,賺錢/賠錢/或者不賠不賺的情況有一些相對應的術語,我們把他寫在下面:

1. In the Money: 
指如果立刻執行選擇權,則 payoff $>0$; (立刻執行會賺,稱 In the Money)
2. Out of the Money: 
指如果立刻執行選擇權,則 payoff $<0$; (立刻執行會賠,稱 Out of the Money)
3. At the Money: 
指如果立刻執行選擇權,則 Payoff $=0$; (立刻執行結果不賺不賠,稱 At the Money)


選擇權的價值 (Option Value)
選擇權價值 (Option Value) = 內在價值 (Intrinsic Value) + 時間價值 (Time Value)

亦即 我們可說
\[
\text{Option Value $\geq$ Intrinsic Value }
\] 其中 內在價格 = 如果現在立刻執行該選擇權所會獲得的 payoff;
時間價格 $>0$ always。

上述關係說明了 時間價格的隱含意義,他指出時間價格 用來表示儘管選擇權在一段時間內是 Out-of-money,但是隨著(未來)時間變動有可能會有上漲/下跌的可能,此可能性會導致儘管現在選擇權可能是處在 Out-of-money的狀態,其 Option Value 仍然是正值。因為人們可能預期未來選擇權的價值可能會上升

有了上述概念,我們可以開始先看一下一般而言,實際上 選擇權 是如何報價的:
下圖為美國芝加哥交易所(www.cboe.com) 對 S&P500 Index 的 American Option 報價 (Bid-Ask price):

NOTE: 
Ask price: 你要購買選擇權所需花費的價格,
Bid price 你要賣出選擇權所得的價格


上圖中,左方綠色部分為 Call option (買權) ,右方則為 Put Option (賣權);且執行價格為 $K=500, 600, 700, ...,1,000$,到期日為 May 2014。

那麼現在問題是,這些Bid-Ask Price 究竟如何得到的呢? 怎麼知道這些價格是否合理??
這些問題將留待下一篇文章在做介紹。
[衍生商品] 淺談選擇權 (1) - Some Properties of Option
[衍生商品] 淺談選擇權 (2) - Put-Call Parity

5/06/2011

[最佳控制] 離散時間 穩態 LQR 控制問題 (1)

延續前篇,這次要介紹的是 Discrete Time Linear Quadratic Regulator in Infinite Horizon 或稱 Steady State LQR。

================
LQR Problem (Infinite Horizon LQR):
考慮離散狀態方程:
\[
x(k+1) = A x(k) + B u(k)
\]其中 $x(k) \in \mathbb{R}^n, A\in \mathbb{R}^{n \times n}, B \in \mathbb{R}^{n \times m}, u(k) \in \mathbb{R}^{m \times 1}$且 $(A,B)$ controllable。
定義 Performance index:
\[
J(u) = \displaystyle \sum_{k=0}^{\infty} x^T(k+1) Q x(k+1) + u^T(k) R u(k)
\] 其中 $Q, R$ 必須滿足 $Q^T = Q, Q \succ 0$, $R^T = R, R \succ 0$。 (亦即 $Q, R$ 必須為 對稱 + 正定 矩陣)

試求出一組最佳控制力序列 $u^*$ 使得成本函數 $J(u)$ 最小。
================

Comment:
讀者須注意到 Infinite Horizon 的 LQR問題要求計算 Performance index 為無窮級數和,此解必須保證收斂。以下定理告訴我們何時 此 Performance index 收斂

Lemma
考慮離散系統 $x(k+1) = A x(k) + B u(k)$,若 $(A,B)$ 可控制,且選 $Q, R >0$ 為正定矩陣,則上述 infinite horizon LQR 問題保證 閉迴路系統 狀態收斂到 $0$ 且 cost 為有界。

Proof: omitted. (see J. B. Rawlings and D. Q. Mayne, "Model Predictive Control: Theory and Design, p. 24", 2009)


現在我們可以開始求解 Infinite Horizon LQR問題:
Solution
回憶 Steady State Bellman Equation,為了符號簡便起見,我們寫成 functional equation 形式,
\[
I(x) = \displaystyle \min_{u \in \Omega} \{J(x,u) + I(f(x,u)) \}
\] 上式中 $J(x,u)$ 為 Branch cost,亦即 $J(x,u) = x^T Q x + u^T R u$ (並非 $\sum_{k=0}^{\infty} (\cdot)...$)

首先我們猜一組解 $I(x) = x^T P x$ 且矩陣 $P$ 為對稱正定矩陣,亦即滿足 $P^T = P, P \succ 0$。我們之後會找到此 $P$ 應該長甚麼樣子。

將猜測的解代入上述的 Steady State Bellman Equation,故現在我們得到
\[
I(x) = \min_{u \in \Omega} \{J(x,u) + I(f(x,u)) \}
\]注意到 $I(f(x,u) = f(x,u)^T P f(x,u) = (Ax+Bu)^TP(Ax+Bu)$,故我們可得
 \[
\begin{array}{l} I(x) = \mathop {\min }\limits_{u \in \Omega } \{ J(x,u) + I(f(x,u))\} \\ \Rightarrow {x^T}Px = \mathop {\min }\limits_u \left\{ {{x^T}Qx + {u^T}Ru + {{\left( {Ax + Bu} \right)}^T}P\left( {Ax + Bu} \right)} \right\}\\ \Rightarrow {x^T}Px = \mathop {\min }\limits_u \left\{ {{x^T}\left( {Q + {A^T}PA} \right)x + 2{x^T}{A^T}PBu + {u^T}Ru + {u^T}{B^T}PBu} \right\} \end{array}
\]透過一階必要條件 FONC: $ \frac{\partial }{{\partial u}} = 0$ 對上式右邊求解
 \[\begin{array}{l} 2{\left( {{x^T}{A^T}PB} \right)^T} + 2Ru + 2{B^T}PBu = 0\\ \Rightarrow {u^*} = - {\left( {R + {B^T}PB} \right)^{ - 1}}{B^T}PAx \end{array}
\]現在將 $u^*$ 代回 $(*)$  可得 \[\begin{array}{l} {x^T}Px = \mathop {\min }\limits_u \left\{ {{x^T}\left( {Q + {A^T}PA} \right)x + 2{x^T}{A^T}PBu + {u^T}Ru + {u^T}{B^T}PBu} \right\}\\ \Rightarrow {x^T}Px = \left\{ {{x^T}\left\{ {Q + {A^T}PA - {A^T}PB{{\left( {R + {B^T}PB} \right)}^{ - 1}}{B^T}PA} \right\}x} \right\} \end{array}
\]比較左右兩邊可得到 $P$ 必須滿足下式: \[P = Q + {A^T}PA - {A^T}PB{\left( {R + {B^T}PB} \right)^{ - 1}}{B^T}PA\] 此式稱為 Discrete Time Algebraic Ricatti Equation (ARE),一般而言,可利用 MATLA 指令 dare(A,B,Q, R) 求解 P。

由於 $u^* = - {\left( {R + {B^T}PB} \right)^{ - 1}}{B^T}PAx$,其中除了 $P$ 未定之外,其餘所需要的參數都已知且皆與跌代時間無關,故此無窮時間LQR問題得到的 最佳控制力為 Time invariant。

現在我們總結如下:求解無窮時間的LQR問題只要做兩個步驟即可
STEP 1: 求解一次 Algebraic Ricatti Equation 得到 $P$ (利用 MATLAB: dare.m 或者徒手計算)
\[
P = Q + {A^T}PA - {A^T}PB{\left( {R + {B^T}PB} \right)^{ - 1}}{B^T}PA
\]STEP2 : 將 $P$ 代入 ${u^*} =  - {\left( {R + {B^T}PB} \right)^{ - 1}}{B^T}PAx$

下面我們看個例子:

Example:
考慮一個離散時間線性系統狀態方程:
\[
x_1(k+1) = x_2(k) \\
x_2(k+1) = x_1(k) + u(k)
\]且考慮 Cost function:
\[
J = \sum_{k=0}^{\infty}2x_1^2(k) + 2x_1(k)x_2(k) + x_2^2(k) + 3u^2(k)
\] 且控制力具有如下形式:
\[
u(k) = K_1 x_1(k) + K_2 x_2(k)

\] 試求 $K_1, K_2$ 使 上述 Cost function 最小:

Solution
首先定義  $x(k) := [x_1(k), x_2(k)]^T$ ,則我們有
\[x\left( {k + 1} \right) = \underbrace {\left[ {\begin{array}{*{20}{c}}
0&1\\
1&0
\end{array}} \right]}_A\left[ {\begin{array}{*{20}{c}}
{{x_1}(k)}\\
{{x_2}(k)}
\end{array}} \right] + \underbrace {\left[ {\begin{array}{*{20}{c}}
0\\
1
\end{array}} \right]}_Bu(k)
\] 與 Cost function
\[\begin{array}{l}
J = \sum\limits_{k = 0}^\infty {(2x_1^2(} k) + 2{x_1}(k){x_2}(k) + x_2^2(k) + 3{u^2}(k))\\
\begin{array}{*{20}{c}}
{}
\end{array} = x{\left( k \right)^T}\underbrace {\left[ {\begin{array}{*{20}{c}}
2&1\\
1&1
\end{array}} \right]}_Qx\left( k \right) + \underbrace 3_R{u^2}(k)
\end{array}
\] 那麼現在此問題變成 Steady-state LQR problem,故由前述討論可知我們有 Optimal feedback control 為
\[
u^*(k) = -(R+B^T P B)^{-1} B^T PA \cdot x(k)

\] 其中 $P$ 滿足 $P=P^T, P \succ 0$ 可由 ARE
\[
P= A^TPA - A^T PB (R+ B^TPB)^{-1}B^TPA+Q

\]利用 MATLAB 指令 dare(A,B,Q,R) 解得 $P = \left[ {\begin{array}{*{20}{c}}
{3.7841}&{1.6815}\\
{1.6815}&{4.4022}
\end{array}} \right]$ 現在將 $P$ 帶回 $u^*$中
\[u\left( k \right) = - \left[ {\begin{array}{*{20}{c}}
{0.5947}&{0.2272}
\end{array}} \right]x\left( k \right)

\]i.e., $K_1 = -05947, K_2 =-0.2272$. $\square$


5/05/2011

[最佳控制] 離散時間 LQR- Finite Time Horizon

這次要介紹 控制理論中一個重要的結果:
 Discrete Time Linear Quadratic Regulator (LQR) in Finite Time Horizon,
中文翻譯為 離散時間線性二次調節器,我們這邊會針對此問題利用 Dynamic Programming 的方法來逐步求解

================
LQR Problem (Finite Horizon):
考慮狀態方程:
\[
x(k+1) = A x(k) + B u(k)
\]其中 $x(k) \in \mathbb{R}^n, A\in \mathbb{R}^{n \times n}, B \in \mathbb{R}^{n \times m}, u(k) \in \mathbb{R}^{m \times 1}$
且 考慮 Performance index:
\[
J(u) = \displaystyle \sum_{k=0}^{N-1} x^T(k+1) Q x(k+1) + u^T(k) R u(k)
\] 其中 $Q, R$ 必須滿足 $Q^T = Q, Q \succ 0$, $R^T = R, R \succ 0$。 (亦即 $Q, R$ 必須為 對稱 + 正定 矩陣)

試求出一組最佳控制力序列 $u(N-1), u(N-2),... u(0)$ 使得成本函數 $J(u)$ 最小。
================

Comment:
1. LQR 顧名思義是其具有系統狀態方程為線性 $x(k+1) = Ax(k)+Bu(k)$ 與  Performance Index 中的項都為二次式。
\[
J(u) = \displaystyle \sum_{k=0}^{N-1} x^T(k+1) Q x(k+1) + u^T(k) R u(k)
\]
2. 上述對於矩陣 $Q, R$ 的對稱與正定假設是必須的 (之後需求在求解最佳控制力序列的時候需要求解反矩陣,故需要這些性質。)

3. 上述 LQR in Finite Horizon的問題所求得的最佳控制力序列為 Time Varying 。(此性質會在下面求解的時候再度強調。)

4. 若 Performance index 考慮 $N \rightarrow \infty$,亦即
\[
J(u) = \displaystyle \sum_{k=0}^{\infty} x^T(k+1) Q x(k+1) + u^T(k) R u(k)
\]則我們說此問題為 LQR in Infinite Horizon 或稱 Steady State LQR。這類問題會在之後再做介紹。(此類問題所得到的最佳控制序列將不再是 Time Varying,且只需求解一次 Algebraic Ricatti Equation 即可獲得最佳控制力序列)



Solution of Discrete Time LQR problem in Finite Horizon 
現在我們開始進行求解。

這邊我們使用 Dynamic Programming 方法來求解上述 LQR問題。回憶 Dynamic Programming Equation 的定義:
\[I\left( {x\left( l \right),N - l} \right) = \mathop {\min }\limits_{u\left( l \right) \in {\Omega _l}} \left\{ {J\left( {x\left( l \right),u\left( l \right)} \right) + I\left( {x\left( {l + 1} \right),N - \left( {l + 1} \right)} \right)} \right\}
\] 考慮 Optimal Cost of one-step-to-go ($l=N-1$):
\[\begin{array}{l}
 \Rightarrow I\left( {x\left( {N - 1} \right),1} \right) = \min \left\{ {J\left( {x\left( {N - 1} \right),u\left( {N - 1} \right)} \right) + I\left( {x\left( N \right),0} \right)} \right\}\\
\begin{array}{*{20}{c}}
{}&{}&{}&{}&{}&{}
\end{array} = \min \left\{ {{x^T}(N)Qx(N) + {u^T}(N - 1)Ru(N - 1)} \right\}
\end{array}
\]將系統狀態方程 $x(k+1) = A x(k) + B u(k) $ 代入:
\[\begin{array}{l}
 \Rightarrow I\left( {x\left( {N - 1} \right),1} \right) = \\
\begin{array}{*{20}{c}}
{}&{}&{}&{}
\end{array}\min \left\{ \begin{array}{l}
{\left[ {Ax\left( {N - 1} \right) + Bu\left( {N - 1} \right)} \right]^T}Q\left[ {Ax\left( {N - 1} \right) + Bu\left( {N - 1} \right)} \right]\\
 + {u^T}(N - 1)Ru(N - 1)
\end{array} \right\}\\
 \Rightarrow I\left( {x\left( {N - 1} \right),1} \right) = \min \left\{ \begin{array}{l}
{x^T}\left( {N - 1} \right){A^T}QAx\left( {N - 1} \right)\\
\begin{array}{*{20}{c}}
{}
\end{array} + 2{x^T}\left( {N - 1} \right){A^T}QBu\left( {N - 1} \right)\\
\begin{array}{*{20}{c}}
{}
\end{array} + {u^T}\left( {N - 1} \right){B^T}QBu\left( {N - 1} \right)\\
\begin{array}{*{20}{c}}
{}
\end{array} + {u^T}(N - 1)Ru(N - 1)
\end{array} \right\}
\end{array}
\] 由一階必要條件 FONC: ${\nabla _{u\left( {N - 1} \right)}}I\left( {x\left( {N - 1} \right),1} \right) = 0$ 可知
\[\begin{array}{l}
2{B^T}{Q^T}Ax\left( {N - 1} \right) + 2{B^T}QBu\left( {N - 1} \right) + 2Ru(N - 1) = 0\\
 \Rightarrow \left[ {R + {B^T}QB} \right]u(N - 1) =  - {B^T}{Q^T}Ax\left( {N - 1} \right)\\
 \Rightarrow u^*(N - 1) =  - \underbrace {{{\left[ {R + {B^T}QB} \right]}^{ - 1}}{B^T}QA}_{K\left( {N - 1} \right)}x\left( {N - 1} \right)
\end{array}
\]接著我們把上述求得的 1-step-to-go 的最佳控制力 $u^*(k)$ 代回 Optimal cost of 1-step-to-go,我們得到 (透過一些代數運算)
\[I\left( {x\left( {N - 1} \right),1} \right) = {x^T}\left( {N - 1} \right)\underbrace {{A^T}\left\{ {Q - QB{{\left[ {R + {B^T}QB} \right]}^{ - 1}}^T{B^T}Q} \right\}A}_{: = M\left( {N - 1} \right)}x\left( {N - 1} \right)
\]在做下一步跌代之前我們先給個 comments

Comments :
1.注意到上式中我們求 ${u^*}(N - 1) =  - {\left[ {R + {B^T}QB} \right]^{ - 1}}{B^T}QAx\left( {N - 1} \right)$ 需要計算反矩陣 ${\left[ {R + {B^T}QB} \right]^{ - 1}}$ ,故需檢驗反矩陣是否存在:不過這個問題可以被證明反矩陣確實存在: (因為 $R$ 為對稱正定矩陣,$B^TQB$ 為對稱半正定矩陣,由FACT: 正定矩陣+ 半正定矩陣 = 正定矩陣,且正定矩陣具有 eigenvalue 全為正,故可推知反矩陣存在。)

2. 上式所求得的最佳控制力 ${u^*}(N - 1) =  - {\left[ {R + {B^T}QB} \right]^{ - 1}}{B^T}QAx\left( {N - 1} \right) =  - K\left( {N - 1} \right)x\left( {N - 1} \right)$ 其中的 $K_{N-1}$ 稱作 Optimal Gain Matrix。此控制力具有回授控制形式 Feedback form。 (類似 $u=-Kx$ 的形式)

3. 最佳控制力儘管具有回授控制 (Feedback form)的形式,但在此問題中為時變得 Gain。此性質在下一步跌代會顯現出來。

4. 當我們有了上述第一步跌代的結果,整個 LQR問題就簡單許多,因為之後的跌代只有 $Q$ 矩陣會改變其餘結果均不變。我們直接看下一步跌代便會發現此性質:

Back to computation :
考慮 Optimal Cost of 2-steps-to-go: ($l=N-2$):
\[\begin{array}{l}
I\left( {x\left( {N - 2} \right),2} \right) = \min \left\{ {J\left( {x\left( {N - 2} \right),u\left( {N - 2} \right)} \right) + I\left( {x\left( {N - 1} \right),1} \right)} \right\}\\
 \Rightarrow I\left( {x\left( {N - 2} \right),2} \right)\\
\begin{array}{*{20}{c}}
{}&{}
\end{array} = \min \left\{ \begin{array}{l}
{x^T}(N - 1)Qx(N - 1) + {u^T}(N - 2)Ru(N - 2)\\
\begin{array}{*{20}{c}}
{}&{}&{}&{}
\end{array} + {x^T}\left( {N - 1} \right)M\left( {N - 1} \right)x\left( {N - 1} \right)
\end{array} \right\}\\
 \Rightarrow I\left( {x\left( {N - 2} \right),2} \right)\\
\begin{array}{*{20}{c}}
{}&{}
\end{array} = \min \left\{ {{x^T}(N - 1)\underbrace {\left[ {Q + M\left( {N - 1} \right)} \right]}_{: = \tilde Q}x\left( {N - 1} \right) + {u^T}(N - 2)Ru(N - 2)} \right\}
\end{array}
\]可以發現上式中只有 $Q$ 變成了 $\tilde{Q}$ 其餘參數均固定。故我們可以馬上寫下對應的最佳控制力 $u^*(N-2)$:
\[{u^*}(N - 2) =  - {\left[ {R + {B^T}\tilde QB} \right]^{ - 1}}{B^T}\tilde QAx\left( {N - 2} \right) = K (N-2) x(N-2)
\]其對應的 Optimal cost of 2 steps to go:
\[I\left( {x\left( {N - 2} \right),2} \right) = {x^T}\left( {N - 2} \right)\underbrace {{A^T}\left\{ {\tilde Q - \tilde QB{{\left[ {R + {B^T}\tilde QB} \right]}^{ - 1}}^T{B^T}\tilde Q} \right\}A}_{: = M\left( {N - 2} \right)}x\left( {N - 2} \right)
\]故我們只需要持續重複上述步驟,即可依序解得 $K(N-3), K(N-4),..., K(0)$ 與 $M(N-3), M(N-4),...,M(0)$ 與 $J^*$


Comment:
注意到 $u^*(N-2)$ 的 Gain $K(N-2)$ 與 $u^*(N-1)$ 的 Gain $K(N-1)$ 並不相同,此說明了之前所敘的時變特性(Time Varying Property)

[最佳化] C^2 函數一階逼近的餘項積分表示

令 $f: \mathbb{R}^m \to \mathbb{R}$ 為 $C^2$-函數。對 $f$ 在 $y$ 附近使用一階泰勒展開: \[ T_y(x) := f(y) + \nabla f(y)^\top (x - y) \] 則其餘項 $R(x,y)$ 訂為 $$R(...